LSAT and Law School Admissions Forum

Get expert LSAT preparation and law school admissions advice from PowerScore Test Preparation.

User avatar
 Dave Killoran
PowerScore Staff
  • PowerScore Staff
  • Posts: 5850
  • Joined: Mar 25, 2011
|
#80197
Complete Question Explanation
(The complete setup for this game can be found here: lsat/viewtopic.php?t=885)

The correct answer choice is (C)

Template #2 proves that answer choice (C) is possible, and therefore correct.

Answer choice (A) is incorrect because when light 1 is on then light 7 must be off, and because light 8 is not on.

Answer choice (B) is incorrect because two lights on the north side must be on.

Answer choice (D) is incorrect because three consecutively numbered lights are on (6, 7, 8).

Answer choice (E) is incorrect because when light 1 is on then light 2 must be off, and also because three consecutively numbered lights are on (4, 5, 6).

Get the most out of your LSAT Prep Plus subscription.

Analyze and track your performance with our Testing and Analytics Package.